¿Qué es exactamente un fotón?

Considere la pregunta, "¿Qué es un fotón?" . Las respuestas dicen, "una partícula elemental" y no mucho más. En realidad no responden a la pregunta. Además, la pregunta está marcada como un duplicado de "¿Qué es exactamente un cuanto de luz?" – las respuestas allí tampoco me dicen qué es un fotón. Tampoco ninguna de las respuestas a esta pregunta mencionada en los comentarios. Cuando busco "fotón" , no puedo encontrar nada útil. Preguntas como "Función de onda de un fotón" parecen prometedoras, pero no dan fruto. Otros dicen cosas como "el fotón es una excitación del campo de fotones". Eso no me dice nada. ni eldescripción de la etiqueta , que dice:

El fotón es el cuanto de los cuatro potenciales electromagnéticos y, por lo tanto, la partícula bosónica sin masa asociada con la fuerza electromagnética, comúnmente también llamada 'partícula de luz'...

Diría que eso es menos que útil porque da la impresión de que los fotones siempre aparecen y vuelan de un lado a otro ejerciendo fuerza. Este mismo concepto también está en el artículo de Wikipedia sobre fotones , pero no es cierto. Como dijo anna , "las partículas virtuales solo existen en las matemáticas del modelo". Entonces, ¿quién puede decirme qué es un fotón real, o referirme a algún tipo de definición informativa autorizada que sea aceptada y en la que confíen los físicos de partículas? Digo todo esto porque creo que es de suma importancia. Si no tenemos una idea clara de lo que realmente es un fotón, carecemos de fundamento. Es como lo que dijo kotozna :

Los fotones parecen ser una de las ideas fundamentales de la mecánica cuántica, por lo que me preocupa que, sin una definición clara o un conjunto de ejemplos concretos, la base para comprender los experimentos cuánticos sea un poco confusa.

Secundo eso, solo que más. ¿Cómo podemos entender la producción de pares si no entendemos qué es el fotón? ¿O el electrón? ¿O el campo electromagnético? ¿O todo lo demás? Todo comienza con el fotón.

Daré una recompensa de 400 puntos a la respuesta menos mala a la pregunta. Una respuesta obtendrá la recompensa, incluso si no me gusta. Y la pregunta es esta:

¿Qué es exactamente un fotón?

La gente puede encontrar útil este artículo; ¿Hay fotones de hecho? SA Rashkovsky
¡Interesante! El autor afirma que muchas cosas que generalmente se cree que requieren fotones discretos se pueden explicar usando ondas de radiación más o menos continuas y átomos discretos, cargas, etc. ¿Sabe si quedan cosas que requieren cuantos discretos de luz, más allá de las que mira?
Eso es fácil Juan. Como les dije en nuestra conversación anterior últimamente, el fotón es simplemente un bucle cerrado (círculo) 2D de flujo electromagnético que viaja a través del espacio con c. Más o menos como un anillo de vórtice. El perímetro 2πr del bucle es la longitud de onda del fotón. El giro del círculo es 1.
Tal vez el fotón real sean los paquetes de energía que absorbimos en el camino.
Falta la palabra «gravedad» en esta página. ¿Se desvía el haz de luz cerca de la masa para un observador independiente? El haz de luz consiste en fotones? ¿Se ha modificado el impulso de los fotones cerca de la masa para un observador independiente? ¿Ha cambiado el impulso de la masa de acuerdo con la conservación del momento para un observador independiente? ¿Es esto posible si el fotón no tiene una trayectoria definida entre la emisión y la absorción?
"... el fotón son los paquetes de energía..." ¿Quieres decir que como el gluón, esa es la forma en que las masas intercambian masas (c es una constante y la dirección es curva: en mi opinión, el intercambio se diría incorrectamente, ya que las masas expulsan fotones) , otras masas los agarran) perdón por ser espontáneo.

Respuestas (21)

La palabra fotón es una de las palabras más confusas y mal utilizadas en la física. Probablemente mucho más que otras palabras en física, se usa con varios significados diferentes y uno solo puede tratar de encontrar cuál es el significado en función de la fuente y el contexto del mensaje.

El fotón que el experimentador de espectroscopia usa para explicar cómo se conectan los espectros a los átomos y las moléculas es un concepto diferente del fotón del que hablan los experimentadores de óptica cuántica cuando explican sus experimentos. Esos son diferentes del fotón del que hablan los experimentadores de alta energía y todavía hay otros fotones de los que hablan los teóricos de alta energía. Probablemente haya aún más variantes (e innumerables modificaciones personales) en uso.

El término fue introducido por GN Lewis en 1926 para el concepto de "átomo de luz":

[...] uno podría haber estado tentado de adoptar la hipótesis de que estamos tratando aquí con un nuevo tipo de átomo, una entidad identificable, increable e indestructible, que actúa como portador de energía radiante y, después de la absorción, persiste como un constituyente esencial del átomo absorbente hasta que luego se envía de nuevo con una nueva cantidad de energía [...]
- "El origen de la palabra "fotón""

Por lo tanto, me tomo la libertad de proponer para este hipotético nuevo átomo, que no es luz pero juega un papel esencial en todo proceso de radiación, el nombre de fotón.
"La conservación de los fotones" (18 de diciembre de 1926)

Hasta donde yo sé, este significado original de la palabra fotón ya no se usa, porque todas las variantes modernas permiten la creación y destrucción de fotones.

El fotón del que suele hablar el experimentador en espectroscopia UV-visible es un objeto que tiene una frecuencia definida v y energía definida h v ; su tamaño y posición son desconocidos, quizás indefinidos; sin embargo, puede ser absorbido y emitido por una molécula.

El fotón del que suele hablar el experimentador en óptica cuántica (estudios de correlación de detección) es un "objeto cuántico" deliberadamente misterioso que es más complicado: no tiene una frecuencia definida, tiene una posición y un tamaño algo definidos, pero puede abarcar todo el aparato experimental y solo parece como una partícula localizada cuando se detecta en un detector de luz.

El fotón del que habla el experimentador de alta energía es una partícula pequeña que no es posible ver en las fotos de las huellas de partículas y sus eventos de dispersión, pero facilita la explicación de la curvatura de las huellas de partículas de materia con un punto de origen común dentro del marco. de la conservación de la energía y el momento (por ejemplo, la aparición de un par de partículas con carga opuesta, o la dispersión de Compton). Este fotón suele tener un momento y una energía definidos (por lo tanto, también una frecuencia definida) y una posición bastante definida, ya que participa en eventos de dispersión bastante localizados.

Los teóricos también usan la palabra fotón con varios significados. El denominador común son las matemáticas utilizadas para describir el campo electromagnético y su interacción con la materia. Ciertos estados cuánticos especiales del campo EM, los llamados estados de Fock, se comportan matemáticamente de una manera que permite usar el lenguaje de "fotones como cosas contables con energía definida". Más precisamente, hay estados del campo EM que se pueden especificar declarando un conjunto infinito de números enteros no negativos. Cuando uno de estos números cambia por uno, esto se describe mediante una forma de hablar como "creación de fotones" o "destrucción de fotones". Esta forma de describir el estado permite calcular fácilmente la energía total del sistema y su distribución de frecuencia. Sin embargo, este tipo de fotón no puede localizarse excepto en todo el sistema.

En el caso general, el estado del campo EM no es de un tipo tan especial, y el número de fotones en sí mismo no es definitivo. Esto significa que el objeto principal de la teoría matemática del campo EM no es un conjunto de partículas puntuales con un número definido de miembros, sino un campo EM continuo. Los fotones son simplemente una forma de hablar útil cuando el campo es de un tipo especial.

Sin embargo, los teóricos todavía hablan mucho de fotones, en parte porque:

  • está bastante arraigado en el currículo y los libros de texto por razones históricas y de inercia;

  • los experimentadores lo usan para describir sus experimentos;

  • en parte porque causa una buena impresión en las personas que leen relatos populares de física; es difícil hablar de manera interesante ψ función o el espacio de Fock, pero es fácil hablar de "partículas de luz";

  • en parte debido a cómo se enseña el método del diagrama de Feynman .

(En el diagrama de Feynman, a menudo se introduce una línea ondulada en el espacio-tiempo como representación de un fotón. Pero estos diagramas son una ayuda de cálculo para la teoría de perturbaciones para ecuaciones de campo complicadas; la línea ondulada en el diagrama de Feynman no representa necesariamente una partícula puntual real que se mueve a través de espacio-tiempo El diagrama, junto con el fotón al que se refiere, es solo una representación gráfica útil de ciertas integrales complicadas).

Nota sobre la necesidad del concepto de fotón

Muchos experimentos famosos que alguna vez se consideraron evidencia de fotones se explicaron más tarde de forma cualitativa o semicuantitativa basándose únicamente en la teoría de las ondas (teoría EM clásica de la luz, a veces con la ecuación de Schroedinger añadida). Estos son, por ejemplo, el efecto fotoeléctrico , la dispersión de Compton , la radiación de cuerpo negro y quizás otros.

Siempre hubo un grupo minoritario de físicos que evitaron por completo el concepto de fotón para este tipo de fenómenos y prefirieron la idea de que las posibilidades de la teoría EM no están agotadas. Consulte estos documentos para conocer los enfoques de la física que no son de fotones:

R. Kidd, J. Ardini, A. Anton, Evolución del fotón moderno, Am. J. física. 57, 27 (1989) http://www.optica.machorro.net/Lecturas/ModernPhoton_AJP000027.pdf

CV Raman, Una derivación clásica del efecto Compton. Revista india de física, 3, 357-369. (1928) http://dspace.rri.res.in/jspui/bitstream/2289/2125/1/1928%20IJP%20V3%20p357-369.pdf

Trevor W. Marshall, Emilio Santos: El mito del fotón, Arxiv (1997) https://arxiv.org/abs/quant-ph/9711046v1

Timothy H. Boyer, Derivación del espectro de radiación de cuerpo negro sin suposiciones cuánticas, Phys. Rev. 182, 1374 (1969) https://dx.doi.org/10.1103/PhysRev.182.1374

Probablemente valga la pena vincularlo al artículo / artículo de opinión "Anti-photon" de Lamb . Desafortunadamente, no tengo un enlace que no sea de pago.
@dmckee: gracias. Encontré el papel . ¡ Porque te referiste a él hace un par de años!
Entonces, en resumen, un fotón no es exactamente.
@JohnDuffield Debería dar me gusta directamente a su comentario , no a la pregunta en sí;)
Antes de Lewis, había una definición diferente de fotón, una vela por metro cuadrado, en un área de un milímetro cuadrado. Consulte la página 173, nota al pie 20, en el artículo de 1920 aquí: books.google.com/… , la unidad ahora se llama troland en.wikipedia.org/wiki/Troland
+1, porque "se usa con varios significados diferentes y uno solo puede intentar encontrar cuál se refiere según la fuente y el contexto del mensaje". los significados múltiples son siempre una fuente de confusión. Es más, el trabajo necesario para llevar al explicador a este punto de vista, realización, punto de partida, puede ser insuperable.
Esta es una definición muy extraña de cómo los experimentadores de alta energía consideran los fotones. Los fotones reales son rayos gamma duros que se observan a través de lluvias electromagnéticas en calorímetros. También se producen en detectores de centelleo y mediante bremstrahlung (cf. curvatura de pista). También hablamos mucho sobre ellos como propagadores integrales de Feynman, y probablemente somos un poco más arrogantes de lo que deberíamos llamarlos "partículas" en ese contexto ;-)
@andybuckley, solo mencioné lo que creo que es una característica distintiva del concepto del experimentador HEP detrás de la palabra fotón: una partícula que no se puede observar directamente, pero es algo que tiene posición, energía e impulso que ayuda a explicar las interacciones de las partículas cargadas entre sí y su comportamiento en campo EM externo (efecto Compton, creación de pares). Por supuesto, hay otros fenómenos como los que mencionaste. Sería genial si pudieras describir con más detalle cuál es el concepto de fotón del experimentador.
Realmente me gusta la respuesta, realmente no me gusta la pregunta. La pregunta se siente plagada de literatura de divulgación científica interpretada como investigación física real. Sigue siendo una pregunta muy válida, pero parece que la respuesta adecuada es "bueno, atrincherarse en un campo". Soy un científico de óptica ultrarrápida / cuántica y me encuentro regularmente con 3 de las "definiciones" anteriores. Para mí, no hay ambigüedad. Siento que después de que los conceptos están bien cimentados, todos interactúan bien juntos. (1/2)
(2/2) Es como tener cebolla. Después de probarlo se sabe si fue hervido, frito, chamuscado, asado o crudo. Pero sigue siendo cebolla. Si estoy leyendo un artículo, los conceptos necesarios se recuerdan fácilmente. Decir fotón para todos ellos no me da ambigüedad. La luz es una cosa extraña que depende MUCHO de cómo se crea, de su fuerza, de su interacción consigo misma y con la materia. Pero todo son fotones, campos EM.
(por supuesto, puede argumentar: "pero también tiene el término "cebolla hervida" o "cebolla asada", que luego se refieren a una cosa específica". Sí, pero luego comienza a ser más difícil rastrear las cosas. Al aprender qué es el fotón y los sabores en los que aparece, amplías tu comprensión y ves las similitudes entre los diferentes campos. Un chef necesita saber cómo cocinar la cebolla en todas sus formas, no solo en una, ¡incluso si se especializa en un tipo en particular!)

El fotón es una construcción que se introdujo para explicar las observaciones experimentales que mostraban que el campo electromagnético se absorbe y se irradia en cuantos. Muchos físicos toman esta construcción como una indicación de que el campo electromagnético consiste en partículas puntuales adimensionales, sin embargo, de este hecho en particular no se puede estar absolutamente seguro. Todas las observaciones experimentales asociadas al campo electromagnético implican necesariamente el proceso de absorción y/o radiación.

Entonces, cuando se trata de una respuesta estrictamente ontológica a la pregunta "¿Qué es un fotón?" tenemos que ser honestos y decir que realmente no sabemos. Es como esas viejas preguntas sobre la esencia de las cosas; pregunta que en realidad nunca podría ser respondida de manera satisfactoria. El camino hacia una mejor comprensión a menudo requiere que uno se sienta cómodo con la incertidumbre.

Los comentarios no son para una discusión extensa; esta conversación se ha movido a chat .
Esta es una respuesta bastante superficial. Hay significados técnicos para la palabra "fotón" en la teoría cuántica de campos. DavidZ insinúa eso aquí y explico la noción de partícula un poco más, por ejemplo , aquí . Además, se puede encontrar en la mayoría de los tratamientos estándar de QED. Sus afirmaciones sobre la ontología son verdaderas, pero no son realmente la respuesta física a la pregunta. Que el autor de la pregunta parezca insatisfecho o ignorante de esos significados técnicos no significa que no debamos explicarlos.
@ACuriousMind, Sí, es cierto que tenemos el tratamiento más técnico del fotón en la teoría cuántica de campos, etc. Sin embargo, me pareció que la persona que hizo la pregunta conoce estos tratamientos y no está satisfecha con ellos. La pregunta parece requerir una respuesta ontológica.
¿No se "inventó" un cuanto de luz para ayudar con la mecánica cuántica? O era al revés...
“El fotón es una construcción que se introdujo para explicar las observaciones experimentales” Esto se puede decir de cualquier producto del intelecto humano. La pregunta es si esta construcción representa una partícula, signifique lo que signifique, o una ficción conveniente.
¿La forma en que los electrones intercambian masa?

Esta es la tabla de partículas elementales utilizada en el modelo estándar de física de partículas, ya sabes, la que se valida continuamente en el LHC a pesar de las esperanzadas búsquedas de extensiones.

tabla de partículas

El modelo estándar de partículas elementales (representación más esquemática), con las tres generaciones de materia, los bosones de calibre en la cuarta columna y el bosón de Higgs en la quinta.

Tenga en cuenta la palabra "partícula" y tenga en cuenta que cada vez que se calcula un proceso físico para dar números para comparar con las medidas experimentales, estas partículas se tratan como partículas puntuales. es decir, en estos diagramas de Feynman para la dispersión de fotones:

compt

El fotón entrante es real, es decir, en capa de masa, 0 masa, energía h v . El vértice es un punto y es la razón por la que los físicos de partículas siguen hablando de partículas puntuales (hasta que tal vez se valide la teoría de cuerdas, y entonces estaremos hablando de partículas de cuerdas). El concepto de un fotón como partícula es tan realista como el concepto de un electrón, y su existencia está validada por el ajuste a los datos de las predicciones del modelo estándar.

Entonces, la respuesta es que el fotón es una partícula en el modelo estándar de la física que se ajusta a las medidas en dimensiones mecánicas cuánticas, es decir, dimensiones acordes con .

Mirarse el ombligo sobre el "significado de un fotón" más que sobre el "significado de un electrón" en un modelo matemático ya no es física, sino metafísica. es decir, las personas transfieren sus prejuicios de creencias sobre la explicación.

Llamamos a un electrón una "partícula" en nuestras configuraciones experimentales porque la huella macroscópica a medida que pasa a través de los detectores es la de una partícula clásica. Lo mismo ocurre con los fotones medidos en los calorímetros del LHC, su "huella" macroscópica es una partícula de masa cero con energía h v y gira uno.

CMS difotón incluso

Este evento de difotón CMS no tiene dudas sobre si la huella es un fotón o no. Es un fotón de la tabla de partículas elementales. Es sólo en los vértices de la interacción donde la indeterminación de la mecánica cuántica es importante.

Usted pregunta:

¿Cómo podemos entender la producción de pares si no entendemos qué es el fotón?

Parece que uno tiene que enfatizar continuamente que las teorías de la física están modelando datos ; no son una proposición metafísica de cómo comenzó el mundo. Tenemos un modelo QFT exitoso para la física de partículas, que describe el comportamiento de las partículas elementales a medida que se registra su huella en los experimentos y predice con éxito nuevos resultados. Eso es todo.

Entendemos los procesos modelados por QFT , la comprensión de la naturaleza de la configuración axiomática de partículas en la tabla pertenece a la metafísica. Suponiendo los postulados de la mecánica cuántica y las partículas de la tabla, podemos modelar las interacciones de las partículas. Es similar a preguntar "¿por qué S tu ( 3 ) × S tu ( 2 ) × tu ( 1 ) La única respuesta es porque el modelo basado en estos supuestos describe los datos de partículas existentes y predice con éxito nuevas configuraciones.

Me gustaría dar el enlace en una publicación de blog de Motl que ayuda a comprender cómo el campo electromagnético clásico emerge de una gran confluencia de fotones. Necesita las matemáticas de la teoría cuántica de campos. Los campos eléctrico y magnético están presentes en la función de onda del fotón, que es una función de número complejo y no es medible, excepto su complejo conjugado al cuadrado, un número real da la densidad de probabilidad de encontrar el fotón en ( X , y , z , t ) .

Es la superposición de las innumerables funciones de onda de fotones lo que construye la onda EM clásica. La frecuencia de la función de onda del fotón individual aparece en los exponentes complejos que la describen. No debería sorprender que sea la misma frecuencia para la probabilidad que la frecuencia en la onda electromagnética clásica que emerge de innumerables fotones de la misma energía (misma frecuencia). Ambas expresiones matemáticas se basan en la estructura de las ecuaciones de Maxwell, el fotón una forma cuantificada, el EM las ecuaciones clásicas.

Sí, y no por nada lo llaman física de partículas.
¿Puedo obtener una idea de lo que se puede ver en la imagen? Parece demasiado complicado para una colisión de dos fotones, incluso si miramos solo las huellas amarillas.
@JanDvorak si observa el enlace sobre la imagen, es la dispersión de protones en el detector CMS y los dos histogramas azules son los dos fotones de alta energía que salen de la interacción, medidos en los calorímetros electromagnéticos. El resto son partículas cargadas de varios tipos y sería necesario el análisis específico del evento para ver si hay algo más interesante allí.
Anna, tengo una pregunta específica para ti porque creo que eres una de las pocas personas que puede responderla. Se relaciona con la imagen dispersa del LHC. Suponiendo que la colisión representada pudiera repetirse IDENTICAMENTE como sucedió, ¿se dispersarían los dos fotones exactamente de la misma manera? Nuevamente, suponga que la colisión podría repetirse de manera absolutamente idéntica.
@Inquisitivo De manera idéntica, solo se puede controlar mediante la dispersión entrante de protón sobre protón, es decir, la misma energía/momento. De esto se trata un experimento del LHC. Dispersiones de entrada idénticas y medición de las partículas creadas salientes. Entonces, el experimento dice que cada evento es diferente, aunque la entrada sea idéntica. El estudio depende del modelo estándar de física de partículas en.wikipedia.org/wiki/Standard_Model que permite estudiar categorías como, por ejemplo , cds.cern.ch/record/1378102 . En el régimen mecánico cuántico la salida es probabilística
El modelo estándar predice las distribuciones de probabilidad de las dispersiones, con datos de experimentos anteriores, y en este momento está muy bien validado.
gracias ana Uno mas. De su respuesta, ¿puedo concluir lo siguiente? Si la colisión se produjera de nuevo y de manera absolutamente idéntica a la primera colisión, los dos fotones podrían, de hecho, salir disparados en diferentes direcciones desde la primera colisión.
@Inquisitivo . Sí. solo hay una probabilidad para cada evento, calculable, con el SM Uno obtiene distribuciones y las compara con las distribuciones pronosticadas.
Gracias por tu tiempo Ana. Para mí eso es increíble si estoy pensando correctamente.
@Inquisitivo Fue la razón por la que se tuvo que inventar la mecánica cuántica como teoría. Todos los datos del microcosmos contradicen nuestras intuiciones intuitivas del mundo clásico.
Anna, tendía a suponer (debido a un conocimiento incompleto de la teoría de Einstein-Cartan) que el electrón podría tener la extensión espacial (dos o tres órdenes de magnitud mayor que la longitud de Planck) requerida de los fermiones en esa teoría, así que, aunque está un poco fuera de tema con respecto a la pregunta de 2016 en cuestión, ¿le parece plausible que la "huella" del electrón en los vértices pueda representar solo el centro de su masa? (Espero obtener su respuesta para que, con el cuadro y los diagramas a mano, pueda eliminar esa posibilidad de mis conjeturas).
@Edouard En lo que respecta a los experimentos, la huella de un electrón en los detectores está limitada por errores experimentales. Es el modelo estándar de la física de partículas que tiene todas las partículas elementales como partículas puntuales en su teoría cuántica de campos. Como el SM tiene mucho éxito en mapear los datos existentes y predecir nuevos, la presente teoría de partículas tiene al electrón y al resto de partículas elementales como partículas puntuales. Cualquier otra teoría tendría que incorporar el modelo estándar para tener la posibilidad de ser más correcta. El límite experimental del radio del electrón es mucho mayor.
( en.wikipedia.org/wiki/Electron ) que la longitud de Planck, por lo que el campo está abierto, siempre que la teoría final se ajuste a los datos ya existentes, lo que hará si incorpora el modelo estándar.
@Edouard Un ejemplo es la teoría de cuerdas, que tiene las simetrías que pueden incorporar el modelo estándar y asume que las partículas son excitaciones de una cuerda, ya no un punto. (Sin embargo, los teóricos no han logrado encontrar uno entre los miles de posibles).

El punto de partida para explicar los fotones desde un punto de vista teórico deberían ser las ecuaciones de Maxwell. En forma covariante, las ecuaciones en el vacío sin fuentes son

m F m v = 0 m ( ϵ m v α β F α β ) = 0
Es bien sabido que la segunda ecuación se verifica automáticamente si F se define en términos del potencial A
F m v = m A v v A m
Las ecuaciones de Maxwell se pueden obtener del Lagrangiano
L = 1 4 F m v F m v
cuando se utilizan las de Euler-Lagrange variando el potencial .

Este Lagrangiano clásico es la base para la formulación de la teoría cuántica de campos. Dado que las ecuaciones de Maxwell definen una teoría de campo clásica, es natural buscar una descripción QFT, y no solo una descripción QM. Sin entrar en una discusión sobre el significado de la cuantización (que sería demasiado matemático-filosófico y no aclararía su pregunta), supongamos que la formulación de un QFT se puede hacer, de manera equivalente, a través de la integral de trayectoria y la cuantización canónica. Sólo hablaré de esto último.

En la cuantización canónica, el potencial A m y su momento conjugado Π m = L ( 0 A m ) convertirse en operadores con valores de campo que actúan sobre algún espacio de Hilbert. Estos operadores están obligados a satisfacer la relación de conmutación

[ A m ( t , X ) , Π v ( t , X ) ] = i η m v d ( X X )

Debido a esta relación, las dos polarizaciones físicas para A se puede expandir en modos normales que deben interpretarse como operadores de aniquilación y creación, a y a . Si el estado de vacío (es decir, el estado de mínima energía de la teoría) es | 0 , entonces los estados a | 0 se denominan estados de 1 fotón. Por tanto, el fotón es la mínima excitación del potencial electromagnético cuántico .

Todo lo anterior considera solo campos electromagnéticos libres. Eso significa que los fotones se propagan para siempre, no pueden ser emitidos ni absorbidos. Esto está claramente en conflicto con la vida real (y es demasiado aburrido).

Volviendo al electromagnetismo clásico, el Lagrangiano para el campo EM con 4 corrientes j que actúa como fuente es

L = 1 4 F m v F m v A m j m
El ejemplo más común es la corriente creada por un fermión cargado (por ejemplo, un electrón o un muón)
j m = i mi ψ ¯ m ψ

Pero este tipo de lagrangianos presenta un gran inconveniente: no sabemos cuantizarlos de forma exacta . * Las cosas se complican con las interacciones. Sólo podemos hablar, con cierto rigor, de estados asintóticos: estados mucho antes o mucho después de cualquier interacción se asemejan a los de los campos libres. Por tanto, el fotón real es la excitación del potencial electromagnético cuántico que en el límite t ± tiende al fotón libre como se definió anteriormente.

Así que sí, en cierto sentido, tienes razón en que no sabemos qué es un fotón. Pero este obstáculo [formal] no nos impide hacer predicciones, como el caso de la producción de pares que les preocupa. El punto clave es que no sabemos lo que sucede durante la interacción, no podemos saberlo y no necesitamos saberlo . Solo necesitamos comparar los estados asintóticos antes y después de la interacción. Para hacer eso, necesitamos realizar alguna aproximación, generalmente una expansión perturbativa (que da como resultado diagramas de Feynman, las mal llamadas "partículas virtuales" y todo eso). La comparación entre los estados de entrada y salida, codificada en el S matriz , es suficiente para predecir tasas de descomposición, secciones transversales y relaciones de ramificación para cualquier proceso que pueda imaginar. Y esos observables son los únicos que podemos medir. En conclusión, las cosas que no puedes definir con precisión son las cosas que no puedes verificar experimentalmente.

Esta respuesta es solo un boceto, una respuesta completa requeriría que escriba un libro sobre el tema. Si quieres saber más, te animo a leer cualquier libro sobre QFT, como Peskin & Schroeder, Weinberg, Srednicki, etc.


* En una teoría interactiva, las ecuaciones clásicas de movimiento no son lineales y no se pueden resolver usando una expansión de Fourier que produce operadores de creación y aniquilación. En la formulación de la integral de trayectoria, solo sabemos cómo resolver integrales gaussianas (es decir, campos libres). Para resolver las integrales de trayectoria para campos que interactúan, todavía necesitamos métodos aproximados como expansiones perturbativas o QFT de celosía. Según Peskin y Schroeder:

No se conocen teorías de campos interactivos exactamente resolubles en más de dos dimensiones del espacio-tiempo, e incluso allí los modelos resolubles implican simetrías especiales y una complicación técnica considerable.


Para más detalles sobre esto, te remito a esta excelente respuesta de ACuriousMind a otra pregunta tuya.

Me gusta más esta respuesta simplemente porque no se detiene en la cruda realidad de que es la interpretación de las matemáticas lo que define estos objetos.
¿Qué quiere decir con "no sabemos cómo cuantificarlos de manera exacta "?
@GennaroTedesco Que en una teoría interactiva, las ecuaciones clásicas de movimiento no son lineales y no se pueden resolver usando una expansión de Fourier que produce operadores de creación y aniquilación. Según Peskin & Schroeder: "No se conocen teorías de campos interactivos exactamente resolubles en más de dos dimensiones del espacio-tiempo, e incluso allí los modelos resolubles implican simetrías especiales y una complicación técnica considerable".
Bueno, no necesitas escribir la cuantización en términos de operadores de creación y aniquilación; puede simplemente escribir la integral de ruta para el Lagrangiano que interactúa. Entonces, eso sería no renormalizable o mal definido o, de lo contrario, estoy de acuerdo, pero conceptualmente también se pueden cuantificar las teorías que interactúan.
@GennaroTedesco En la formulación de la integral de ruta, solo sabemos cómo resolver integrales gaussianas (es decir, campos libres). Para resolver las integrales de trayectoria para campos que interactúan, todavía necesitamos métodos aproximados como expansiones perturbativas o QFT de celosía.
Estoy totalmente de acuerdo en que en realidad no puedes calcular la integral de ruta, seguro.
Bosoneando: No comparto tu sentimiento: "no sabemos qué es un fotón... no sabemos qué sucede durante la interacción, no podemos saberlo y no necesitamos saberlo".
Bosoneando En mi forma asintóticamente menos experta, comparto mucho su sentimiento (repetido aquí arriba por John Duffield) como se describe en mi PDF al que se hace referencia en mi comentario final-respuesta a @Inquisitive más abajo.

Respuesta ontológica

No hay una respuesta corta.

El fotón es exactamente lo que obtienes cuando estudias todo nuestro conocimiento sobre él en forma de teorías matemáticas, la mayoría de las cuales tienen la gran Q en sus nombres. Y luego, probablemente, algunos más que aún no hemos encontrado. No hay atajos.

Justificación, referencia

Esta es, a todos los efectos, una respuesta de escape. Una pregunta que pregunte "¿cómo se comporta un fotón?", "¿qué sabemos sobre las interacciones de un fotón con XXX?", etc. sería fácil (más o menos) de responder. Pero sugiero que la pregunta "¿qué es un fotón?" (en el sentido de "¿qué es realmente , dejando de lado todas las matemáticas?") no puede tener ninguna respuesta significativa, al igual que la pregunta "¿qué es un XXX?" (donde XXX es cualquier partícula o campo del Modelo Estándar) no tiene significado.

En lugar de escribir mucho, sugiero la entrevista de Feynman, Richard P Feynman - FUN TO IMAGINE (full) ; la parte relevante para esta respuesta (" verdadero significado de las cosas") va desde 01:03:00 hasta el final (resumen: incluso si tenemos las teorías correctas sobre las partículas y, por lo tanto, podemos explicar sus efectos, todavía no tenemos forma para explicarlos en términos cotidianos/prácticos sin las matemáticas, y nunca habrá una, ya que no hay leyes "mundanas" subyacentes). También en parte, con respecto a las respuestas cortas y fáciles, una parte que comienza en 17:20 (resumen: es difícil describir algo completamente en una "resolución" particular, va cada vez más profundo; como dije, solo parcialmente relacionado, pero bastante perspicaz todavía).

EDITAR: se agregaron resúmenes.

"En lugar de escribir mucho". Aún así, un resumen estaría en orden.
Agregado, @PeterMortensen.

Se necesitan muchas páginas para responder realmente a la pregunta de qué es un fotón, y diferentes expertos dan respuestas diferentes. Esto se puede ver en una interesante colección de artículos que explican diferentes puntos de vista actuales:

La naturaleza de la luz: ¿Qué es un fotón? Noticias de óptica y fotónica, octubre de 2003

Mis preguntas frecuentes sobre física teórica contienen una entrada con el título "¿Qué es un fotón?". Aquí hay un breve extracto; pero para responder a la pregunta con cierta profundidad, es necesario leer la entrada de Preguntas frecuentes. Desde el principio:

De acuerdo con la electrodinámica cuántica, la teoría verificada con mayor precisión en la física, un fotón es una excitación de una sola partícula del campo electromagnético cuántico libre. Más formalmente, es un estado del campo electromagnético libre que es un estado propio del operador de número de fotones con valor propio 1.

Los estados puros del campo electromagnético cuántico libre son elementos de un espacio de Fock construido a partir de estados de 1 fotón. Un vector de estado general de n fotones es una combinación lineal arbitraria de productos tensoriales de n vectores de estado de 1 fotón; y un estado puro general del campo electromagnético cuántico libre es una suma de vectores de estado de n fotones, uno para cada n. Si solo contribuye el término de fotón 0, tenemos el estado oscuro, generalmente llamado vacío; si solo contribuye el término de 1 fotón, tenemos un solo fotón.

Un solo fotón tiene los mismos grados de libertad que un campo de radiación de vacío clásico. Su forma se caracteriza por un 4-potencial A(x) real arbitrario distinto de cero que satisface las ecuaciones libres de Maxwell, que en el calibre de Lorentz toma la forma A ( X ) = 0 , expresando la masa cero y la transversalidad de los fotones. Por lo tanto, para cada tal A hay un estado de fotón puro correspondiente |A>. Aquí A(x) no es un operador de campo sino una amplitud de fotones; los fotones cuya amplitud difiere en un factor de fase independiente de x son los mismos.

Y casi al final:

La conversación sobre fotones generalmente se hace de manera inconsistente; casi todo lo que se dice en la literatura sobre los fotones debe tomarse con pinzas. Incluso hay gente como el ganador del premio Nobel Willis E. Lamb (el descubridor del cambio de Lamb) que sostiene que los fotones no existen. Ver hacia el final de
http://web.archive.org/web/20040203032630/www.aro.army.mil/phys/proceed.htm La referencia mencionada allí al final apareció como WE Lamb, Jr., Anti-Photon , Física Aplicada B 60 (1995), 77-84. Esto, junto con la otra referencia mencionada por Lamb, está reimpreso en WE Lamb, Jr., The interpretation of quantum mechanics, Rinton Press, Princeton 2001.

Creo que la interpretación más adecuada de un fotón 'observado' como se usa en la práctica (en contraste con el fotón definido formalmente como se mencionó anteriormente) es como un estado coherente de baja intensidad, cortado arbitrariamente en segmentos de tiempo que transportan una energía de h v = ω , la energía de un fotón en frecuencia nu y frecuencia angular omega. Dicho estado consiste principalmente en el vacío (que no es directamente observable, por lo que generalmente se puede despreciar), y las contribuciones de los estados multifotónicos son insignificantes en comparación con la contribución de un solo fotón. Con tal noción de fotón, la mayoría de los experimentos reales realizados tienen sentido, aunque no explica la aleatoriedad cuántica del proceso de detección (que proviene de los electrones cuantificados en el detector).

Vea también las diapositivas de mis conferencias aquí y aquí .

Para mí, las otras partículas elementales son igualmente misteriosas. Esto se debe a su naturaleza no intuitiva.

ontología ingenua del mundo

Los humanos crecemos en un mundo con objetos . Estos objetos tienen masa y volumen, tienen límites discretos. Nuestros cerebros suelen considerar los objetos como cosas separadas , y cada cosa puede, por ejemplo, ser captada e inspeccionada de otro modo por los sentidos. En consecuencia, las cosas se denotan con nombres ordinarios.

Luego están los fluidos, que son casi como cosas pero no del todo. Tienen masa y volumen, y se puede interactuar con ellos a través de los sentidos (aunque darse cuenta de que el aire no es vacío al notar la resistencia del aire es un salto de comprensión no trivial), pero no están separados. Se fusionan entre sí y se pueden dividir arbitrariamente, a diferencia de las cosas , por lo que nunca se puede saber exactamente qué es un fluido. En consecuencia, los conocemos por sustantivos incontables, y entendemos que en última instancia tiene sentido pensar en partes de un único fluido universal . El fluido universal se convierte en la única cosa , ya que otras subdivisiones son discutibles, excepto en el caso especial de gotas o cantidades secuestradas en recipientes, donde ella cosidad se está imponiendo artificialmente (es decir, puede fingir que el café y la leche son cosas separadas siempre que estén en sus tazas separadas, pero en el momento en que permite que entren en contacto, esta simulación se desmorona).

Todo lo demás lo consideramos fenómeno . Por ejemplo, el fuego es algo que sucede: no es algo que puedas recoger y manipular (solo puedes recoger el combustible, y el fuego se adhiere extrañamente a él), ni es algo que puedas atrapar en contenedores y subdividir. o combinar (aunque el combustible puede ser una cosa o un fluido, y puede operarse, con el fuego a veces acompañando el viaje). Lo mismo ocurre con el sonido, la luz, la temperatura y conceptos similares.

Por supuesto, ahora sabemos que el fuego es solo plasma, es decir. un fluido que se "descompone" en otra cosa muy rápidamente y que, de hecho, es posible atraparlo con el recipiente exótico adecuado. Por lo tanto, la imaginación puede verse obligada a aceptar el fuego como un fluido , aunque en la vida cotidiana todavía aparece como un fenómeno , por lo que no es una cosa verdaderamente intuitiva.

La teoría del átomo

Cuando surgió la teoría del átomo, estoy seguro de que los griegos en algún momento esperaban que los "átomos" tuvieran cierta masa, forma y tamaño, es decir. ser cosas . Por lo tanto, se emplea un ingenioso truco conceptual: para el ojo descuidado, la arena parece un fluido , ya que cantidades de ella parecen fusionarse y dividirse libremente (tenga en cuenta que la arena es incontable). Pero en una inspección más cercana, la arena es solo un montón de pequeños objetos, que en sí mismos son claramente cosas .

Entonces me di cuenta de que el mundo tendría sentido si todo fuera una especie de arena formada por muchas partículas diminutas, que son en sí mismas cosas. Esto es bueno porque une fluidos y cosas: el fluido es solo una clase aparente, y en el fondo son todas las cosas, lo cual es bueno. Las cosas son muy intuitivas para nosotros los humanos, y puede ser más fácil razonar sobre un montón de millones de cosas diminutas que sobre un solo líquido debido a lo extraños que son los fluidos. Afortunadamente, resultó que tanto los fluidos como las cosas estaban compuestos de partículas, y estas partículas parecen cosas de buena fe .

Aquí comienzan las advertencias. Las moléculas no son del todo sólidos newtonianos. Se comportan casi como ellos: por ejemplo, pueden tener masa y volumen. Casi todos se pueden romper, pero si consideras la regla muy rígida de romper una molécula versus romper una roca, ya comienza a parecer divertido. Tienen un límite y chocan entre sí... Pero ten cuidado de no golpearlos demasiado fuerte, o se fusionan extrañamente (a diferencia de las rocas). Pero la peor parte es el límite, que es solo un límite falso: el radio de Van der Waals no es un delimitador binario "puede/no puede pasar", sino que es una consecuencia de una ecuación de fuerza continua. En realidad, no es mucho más difícil estar ligeramente dentro de la molécula que estar ligeramente fuera . Comparar estar ligeramente adentrode una roca - imposible.

Aparte, creo que es interesante que los griegos propusieron una teoría del átomo en lugar de una teoría del fluido, donde todos los objetos sólidos son de hecho fluidos en algún estado temporal de rigidez. Las diversas teorías de los elementos vienen a la mente, pero no hacen las observaciones físicas correctas: uno podría observar que piezas discretas de hierro se pueden derretir y combinar sin problemas, y luego concluir que seguramente debe ser posible derretir cualquier cosa , por lo tanto sólo hay cosas aparentes, y todo es esencialmente un fluido. Tal vez sea porque esta teoría de los fluidos hace que el mundo sea más confuso, no menos.

Partículas subatómicas

Moléculas, como se vio después, eran simplemente pequeñas agregaciones de cosas. ¿Seguramente cuando se rompe una cosa, el resultado debe ser cosas más pequeñas ? Pronto nos enteramos de los átomos y luego de las partes del átomo. Aquí es donde nos detenemos, ya que, que yo sepa, ninguna de las partículas elementales se sabe que sea divisible en otros constituyentes. Los fotones son una de esas partículas elementales.

La apariencia de una cosa puede mantenerse para los átomos y las moléculas a través de dispositivos como el volumen de Van der Waals. Para las partículas elementales, esta pretensión es inútil. Como se ha demostrado una y otra vez, las partículas elementales no solo no tienen volumen, sino que descaradamente no tienen volumen: si lo tuvieran, la física claramente no funciona, y obtienes cosas como "superficies" de electrones girando Más rápido que la velocidad de la luz. Entonces se hizo la observación de que el mundo tendría sentido si solo estas partículas fueran puntos.

Por supuesto, nadie sabe realmente qué es una masa puntual. Nadie ha visto nunca algo así (bueno, excepto lo que deseamos bautizar como masa puntual en primer lugar). Sus implicaciones parecen extrañas: por ejemplo, su densidad es infinita y, en teoría, todo el universo podría comprimirse en un solo punto. Si las partículas hubieran sido cosas , tales locuras estarían cómodamente excluidas: las rocas no se pueden apretar arbitrariamente, ni siquiera con una fuerza infinita.

No ser comprimible, por cierto, es otra propiedad de las cosas . Incluso las cosas blandas como las esponjas resultan tener bolsas de aire en ellas. Una vez que se exprimen todos los agujeros, una cosa no se puede comprimir más: los líquidos respetan cortésmente este principio de labios para afuera, aunque se puede decir (por ejemplo, por el experimento del agua en una jeringa) que sus corazones realmente no están en él, y a los gases simplemente no podría importarles menos, otra forma en la que los fluidos son extraños y diferentes a las cosas . O al menos, para un observador ingenuo sin acceso a las energías extremas que requieren nuestros experimentos físicos modernos.

El nivel subatómico es donde la intuición se rompe por completo. Puedes crear analogías , como cuerdas y ollas de agua, pero nunca puedes imaginar realmente cómo es una partícula en términos de objetos de la vida cotidiana. El Universo nos ha jugado un truco muy cruel, en el sentido de que es de una manera , pero es tal que en el nivel macro en el que necesariamente comenzamos a entenderlo, es completamente de otra manera , sin apariencia de uno . manera de ser visto. Entonces estamos condenados a crecer esperando y acostumbrándonos de otra manera ., solo para tomar Física 201 en la universidad y descubrir que todo lo que sabemos es una ilusión y ninguna intuición es posible para la verdadera naturaleza del mundo. La intuición, en efecto, es una comprensión basada en la experiencia: ¿Quién puede experimentar lo subatómico? En el mejor de los casos podemos experimentar aparatos experimentales .

El enfoque de arriba hacia abajo para comprender el Universo falla, y falla precisamente en el nivel subatómico.

La versión de abajo hacia arriba

Uno puede debatir el verdadero significado de la intuición, pero creo que se puede restaurar algo de cordura al comenzar de nuevo y arreglar todo. Podemos olvidarnos de todo el bagaje ingenuo sobre las cosas y los fluidos , hacer borrón y cuenta nueva y empezar con la verdad fundamental de que en el mundo hay partículas . Las partículas tienen impulso, son puntos, interactúan entre sí y con el vacío de ciertas formas descritas por la mecánica cuántica. Son elementales y no están hechos de unidades más pequeñas. Los fotones, entonces, son una de esas partículas, con propiedades específicas descritas en otra parte (no las repetiré, ya que usted dijo explícitamente en su pregunta que estas descripciones no son lo que desea).

Cuando realmente un gran número de partículas actúan juntas, a nivel macro se producen algunos fenómenos extraños, como "volúmenes" y "transiciones de estado". Realmente no puedes intuir estas rarezas a partir de nuestro conocimiento de las partículas . Pero lógicamente , es decir. si sigue las matemáticas, sabrá que es una consecuencia simple y directa, aunque no intuitiva.

Desafortunadamente, esta intuición de abajo hacia arriba no es muy útil. Toda nuestra vida cotidiana se refiere a macrofenómenos. Muchas de las cosas interesantes en el universo (básicamente, todas las disciplinas además de la física subatómica) son de escala macro. Uno espera que después de aprender física, el mundo se vuelva más fácil de entender, pero aprender la intuición de abajo hacia arriba solo hace que todo sea más difícil. Sospecho que incluso la física subatómica no se hace mucho más fácil, ya que todo el trabajo real se hace con las matemáticas, no con la intuición.

Entonces, en conclusión, la pregunta no puede ser respondida satisfactoriamente. Hay dos formas de entender una pregunta como "qué es una X":

  • "Dime las propiedades más destacadas de X": para el fotón, no faltan varios textos, e incluso en este sitio hay respuestas perfectamente útiles que te han faltado.
  • "Ayúdame a comprender intuitivamente X": Como dije, ninguna intuición es posible sin derribar toda la intuición que has construido a lo largo de tu vida. Si lo derriba, la intuición que se puede obtener no es satisfactoria y solo sirve para causarle dolor de cabeza.

Pero dicho esto, un fotón es una partícula elemental. Se comporta como si fuera un punto. Tiene impulso y se mueve a la velocidad de la luz (lo que implica que no puede detenerse). Tiene una onda electromagnética asociada. La energía transportada por esta onda está cuantificada. El fotón puede interactuar con otras moléculas, por ejemplo, al ser absorbido y emitido; con suficiente energía puedes crearlos "desde cero", y parecen llevar siempre paquetes de energía con ellos.

Uno puede preguntarse si todas las partículas son meramente alguna forma o disposición de fragmentos discretos de energía, que cuando se agregan de cierta manera, conducen a la apariencia de masas puntuales (¿o debería decir "masas"?) y partículas, y si las unidades individuales de energía de Planck son realmente la base de todo lo demás en el universo, y quizás el fotón está muy cerca de cómo se ven estas "energías" "por sí mismas". Tal vez esto se acerque más a lo que estabas preguntando, pero en este punto estoy firmemente en territorio ombligo, así que me detendré aquí.

+1 No estoy seguro de que esto responda directamente a la pregunta, pero definitivamente implica una crítica valiosa y muy interesante de la demanda implícita de "intuición" en la pregunta del OP. Yo mismo no arriesgué una respuesta porque las conclusiones a las que he llegado me colocan firmemente en la categoría de no poder decirle nada al OP (según su definición). Me gustó especialmente tu clara descripción de nuestra tendencia innata a buscar la "cosidad". Gran parte de nuestro pensamiento son simplemente comportamientos evolucionados que reflejan nuestra historia evolutiva, y no necesariamente útiles en los mundos que ahora exploramos como físicos.
Incidentalmente, y curiosamente, aunque a veces hay formas en las que podemos reestablecer el contacto con las nociones de "cosidad" que parecemos programados para amar a través de formas bastante abstractas: hablo de la noción de Wigner de una partícula como los subespacios "atómicos" de un sistema. espacio cuántico de estados que se dejan invariantes por una representación irreducible del grupo de Poincaré. Estas no son "cosas" como estamos programados para concebirlas, pero, detrás de un velo bastante delgado de abstracción, tienen la mayoría de las propiedades que describe en su primera sección que reconoceríamos de manera innata como una "cosa".
Quizás a los griegos se les ocurrieron los átomos porque tenían muchos problemas con el infinito. Por ejemplo, las paradojas de Zeno. Los átomos son una forma de evitar que un fluido sea infinitamente divisible.

¿Quién puede decirme qué es un fotón real? ¿O referirme a algún tipo de definición informativa autorizada que sea aceptada y confiable por los físicos de partículas? Digo todo esto porque creo que es de suma importancia. Si no tenemos una idea clara de qué es realmente un fotón, carecemos de fundamento...

¿Cómo podemos entender la producción de pares si no entendemos qué es el fotón? ¿O el electrón? ¿O el campo electromagnético? ¿O todo lo demás? Todo comienza con el fotón.

Creo que esto es realmente un problema filosófico, no un problema de física. Ninguna respuesta te satisfará, porque estás haciendo una pregunta que es imposible responder con carácter definitivo: ¿Cuál es la esencia de una cosa?

Exactamente el mismo problema existe con cada concepto del pensamiento humano, no solo en la ciencia (¿Qué es la energía? ¿Qué es el tiempo? ¿Qué es el color? ¿Qué es la conciencia?...) sino también en las humanidades (¿Qué es el amor? ¿Qué es la belleza? ? ¿Qué es la felicidad?...). En cada caso, cuanto más tratamos de definir algo, más elusivo se vuelve, menos entendemos realmente cuál es su esencia. Y cuando creemos que lo comprendemos, surge una nueva propiedad que vuelve a desordenar nuestra comprensión.

Estoy de acuerdo con AnoE (quizás porque soy discípulo de Richard Feynman) en que las cosas solo pueden entenderse como la suma de sus propiedades, sus interrelaciones con otras cosas .

En la vida no es necesario saber qué es el amor para experimentarlo, ni saber qué es la justicia para actuar con justicia o reconocer la injusticia. La única definición que podemos dar es resumir nuestra experiencia de una cosa en uno o más "modelos" idealizados que aíslan las características que consideramos "esenciales".

De la misma manera, no es necesario tener una definición definitiva de un fotón como base sólida antes de que podamos estudiar la luz o desarrollar teorías poderosas como QED. Una definición o modelo de trabajo es adecuado, uno que nos permita identificar y estar de acuerdo sobre la experiencia común y las propiedades que estamos investigando.

La historia de la ciencia muestra que los conceptos que usamos se refinan gradualmente durante décadas o siglos, en particular la pregunta "¿Qué es la luz?" Esta falta de definición definitiva no nos ha impedido desarrollar teorías elaboradas como QED y la Relatividad General que nos permiten predecir con una precisión asombrosa y ampliar nuestra comprensión de cómo funciona el universo.

"Fotón", "electrón" y "campo magnético" son solo nuestros modelos de las cosas que encontramos en el universo, para ayudarnos a predecir y encontrar relaciones entre las cosas. Como dice Elias, estos modelos son, necesariamente, conceptos aproximados . No son lo que realmente existe. Es inevitable que cambien a medida que refinamos nuestras aproximaciones para tratar de encajar nuevas propiedades, nuevas observaciones, en el marco de nuestra comprensión, nuestras teorías.

conceptos aproximados ? de que ? de alguna realidad ? No estoy de acuerdo a menos que diga que son aproximaciones del siguiente paso del conocimiento humano. Son conceptos en una teoría que responde a un conjunto de preguntas. Eso es todo y es mucho.
Lo que quiero decir es que un modelo de trabajo no tiene que ser realista en todos sus atributos. Estoy de acuerdo en que el modelo es particular de una teoría. Pero creo que solo podemos clasificar los modelos como útiles , no como verdaderos . Maxwell desarrolló sus ecuaciones a partir de un modelo de vórtices moleculares que hoy en día parecen ridículos, pero ese modelo funcionó para él. Creo que esto está de acuerdo con su definición final: el fotón es un concepto en una teoría. Para especificar el concepto, necesitamos especificar la teoría. Al igual que con Maxwell, cuando la teoría está completa, podemos descartar el modelo.
¿Podría describirse esto como el proceso infinito de condensación de un concepto en el espacio del lenguaje humano, que por naturaleza es un conjunto de aproximaciones en evolución?
@danielAzuelos: bueno, pero la convergencia de aproximaciones a un ideal involucra el concepto de realidad, eso no ayuda mucho y es un poco difícil de manejar. La cautela es una virtud en este asunto...

Estoy molesto por las definiciones de fotón como se describe en la pregunta. No es que se equivoquen, sino que me engañaron casi como si me impidieran entender qué es un fotón. A continuación se muestra lo que pienso ahora. Eso, por supuesto, no es física nueva, y cada interpretación es subjetiva. Examinaré esto introduciendo algunas antítesis.

1. Los fotones no son discretos

Términos como 'partícula', 'cuanto de luz' o 'unidad de intercambio de energía' hacen creer que los fotones son algo discreto y repentino. La segunda cuantización complementa esta idea. Por ejemplo, en la segunda cuantización, el hamiltoniano de un solo estado (por ejemplo, una onda estacionaria particular en una cavidad) se puede escribir como

H = ω ( a a + 1 / 2 )

Este es también el hamiltoniano de un oscilador armónico. En consecuencia, podemos escribir fácilmente la 'función de onda' de este estado como Ψ ( q ) y hamiltoniano con energía cinética clásica como pags 2 y energía potencial como q 2 términos. Podemos escribir esta función de onda como una combinación lineal,

Ψ ( q , t ) = norte C norte ( t ) ψ norte ( q ) ,

y nos damos cuenta de que la dinámica de los fotones no es tan diferente de la dinámica de los electrones. En medio de la dinámica cuántica (es decir, entre mediciones), puede haber cualquier tipo de paquete de ondas descrito por Ψ ( q , t ) o los coeficientes de combinación lineal C norte ( t ) . Por lo tanto, el número de fotones no es discreto y no se intercambian instantáneamente en cantidades discretas.

En cambio, todo lo que es, es el campo, y está sujeto a la evolución de onda cuántica típica. Este campo se acopla a la materia.

2. La cuantificación no es única

Discutamos los dos modos transversales de un fotón que se propaga (en realidad hay dos más, longitudinal y similar a la energía, pero eso está fuera del alcance). A menudo se dice que un fotón tiene un momento angular de ± , que corresponde a partículas de luz polarizadas circularmente. Esto conduce a una representación similar a un espinor para un fotón.

[ Ψ L ( q ) Ψ R ( q ) ]

Sin embargo, en algunas aplicaciones, es mejor analizar solo fotones polarizados linealmente ( Ψ X , y ( q ) = 1 2 ( Ψ L ( q ) ± i Ψ R ( q ) ) ). Ahora, es fácil ver que, al igual que el espín de los electrones, uno ha elegido solo un marco de referencia preferido y no hay nada extremadamente especial en la elección de estas coordenadas discretas. (Por supuesto, hay algo especial en la elección de coordenadas: la intuición física para describir bien un problema). Pero, de hecho, creo que incluso la transversalidad de una polarización es una elección de referencia.

3. El colapso de la función de onda crea la aparente discreción.

Digamos que una molécula de colorante se excita en nuestro ojo-receptor, y posteriormente cambia su forma, y ​​se transmite un impulso nervioso. Este proceso se asemeja a una medida cuántica, ya que involucra tantos grados de libertad descontrolados en altas temperaturas, y ocurre un fenómeno llamado decoherencia. Por lo tanto, si la función de onda de fotones fue previamente ( 1 2 | 1 > + mi i θ 1 2 | 0 > , la función de onda efectiva (integrando los grados de libertad macroscópicos) está en un estado discreto con probabilidades dadas por sus amplitudes. Es por eso que los fotones se pueden ver y escuchar como clics. Como un grano de sal, es el colapso de la función de onda lo que produce el sonido :)

4. Los fotones de campo lejano y de campo cercano son diferentes

A menudo se dice que un fotón tiene una energía y un momento definidos que deben conservarse (es decir, sigue la relación de dispersión mi = k y un fotón que llega al detector siempre tiene esto (E,k). Pero, por ejemplo, hay cristales fotónicos, donde las energías de los fotones tienen bandas prohibidas y los fotones parecen tener masas (relaciones de dispersión no lineales). Una vez más, se pueden cuantificar las ecuaciones de Maxwell en un cristal fotónico mediante alguna elección de estados y asignar partículas a estos estados. Aquí también se puede hablar de fotones, e incluso decir que tienen masa ya que sus ecuaciones de movimiento se comportan como si tuvieran masa.

Sin embargo, dado que la medición generalmente se realiza en campo lejano, donde los fotones son asintóticamente libres, se miden fotones como mi = k .

5. Los modos no son únicos

Ahora imagina más modos que solo uno antes. La función de onda es ahora Ψ ( q 1 , q 2 q norte ) . Ahora imagine crear una combinación lineal de estos modos q i = j A i j q j para localizarlos tanto como sea posible. De hecho, vamos a localizar en la medida en que un modo q corresponderá a una ubicación particular en el espacio. Ahora tienes una 'unción de onda' de un fotón, lo que da una amplitud de probabilidad del campo de fotones en diferentes posiciones del espacio.

Ψ ( r 1 , r 2 , r norte )

Limitándonos a N coordenadas que describen un fotón aproximadamente alrededor de posiciones ( r 1 r norte ), hemos impuesto efectivamente un corte de energía a nuestras ecuaciones y todo está bien.

Ahora imagine extender este proceso a un límite continuo (lejos de ser trivial) y encender la interacción luz-materia, y nos hemos encontrado con el problema de la renormalización y todas las cosas realmente duras y duras.

Dado todo eso, uno quiere por razones prácticas y por el bien de las intuiciones físicas volver a la segunda cuantización y hablar sobre un fotón en el modo 15. En otras palabras, la segunda cuantización y la conversación sobre partículas como excitaciones de osciladores armónicos es simplemente instrumentos creados por y para la intuición física. Pero si uno quiere entender qué es un fotón, necesita ir debajo del capó.

Estás confundiendo entre tantas cosas diferentes, mezclando QM y QFT sobre todo (¿y con el hecho de que la cuantificación no es única?). ¿Qué es tan insatisfactorio en la descripción clásica QFT de la cuantificación del campo electromagnético?
@GennaroTedesco Mezclé QM y QFT aquí porque quería mostrar que los conceptos básicos de los campos cuánticos se pueden entender bastante bien con QM normal. (Pero básicamente, la única referencia que hago a QFT es sobre la renormalización, el resto es una forma de QED de cavidad). Pero puedo estar de acuerdo en que tal vez fue confuso :) Y tal vez también haya algunas confusiones, me encantaría si podrías elaborar. Sin embargo, esto es hasta donde llega mi comprensión actual de los fotones y hasta ahora he encontrado que esto es lo más satisfactorio. Cambiaré de vista tan pronto como recopile más conocimiento.
La cuestión es que no hay ningún fotón en QM; solo emerge como portador de fuerza del campo electromagnético en QFT (cualquiera que sea el procedimiento de cuantización que quieras elegir), lo que no debe confundirse con la discretización de la energía en procesos como absorción, radiación y todo lo demás. Entonces, si la descripción de QFT está bien planteada o no, es otro tipo de pregunta, pero no surgen contradicciones ni confusiones de QM solo porque no hay ningún fotón involucrado.
@GennaroTedesco Estaba haciendo un tratamiento QM a los grados de libertad del campo electromagnético. La evolución cuántica que describo puede ser un ejemplo como tal en la ecuación. 21 de este artículo nano-bio.ehu.es/files/articles/… Ahora, los acoplamientos se dan ad hoc en esta formulación, y estoy de acuerdo en que se debe hacer un QFT adecuado con (más allá de mí) renormalizaciones para obtenerlos correctamente. Entonces, lo que hice fue usar un sistema QED de cavidad modelo para simplificar la descripción de un fotón. Creo que su línea QM vs. QFT sin fotones en QM es demasiado estricta.

Voy a comenzar mi respuesta refiriéndome a otra diferente: ¿Qué es más fundamental, campos o partículas ?

El fotón es realmente solo un caso especial de lo que se describe en esta respuesta. Citando a Daniel Sank:

Considere una cuerda de violín que tiene un conjunto de modos de vibración. Si desea especificar el estado de la cuerda, enumera los modos y especifica la amplitud de cada uno, por ejemplo, con una serie de Fourier

desplazamiento de cuerda ( X ) = modo  norte = 0 C norte [forma de modo  norte ] ( X ) .

Los modos de vibración son como los estados propios cuánticos, y las amplitudes C norte son como el número de partículas en cada estado. Con esa analogía, la primera notación de cuantización, en la que indexas las partículas y especificas el estado de cada una, es como indexar las unidades de amplitud y especificar el modo de cada una. Eso es obviamente al revés. En particular, ahora ve por qué las partículas son indistinguibles. Si una partícula es solo una unidad de excitación de un estado cuántico, al igual que las unidades de amplitud de una cuerda vibrante, no tiene ningún sentido decir que la partícula tiene identidad. Todas las unidades de excitación son iguales porque son solo construcciones matemáticas para realizar un seguimiento de cuán excitado está un modo en particular.

Una mejor manera de especificar un estado cuántico es hacer una lista de cada estado posible y decir qué tan excitado está.

Un fotón es exactamente eso: una unidad de excitación (1) de un modo del campo electromagnético.


El principal problema con el fotón es que la gente trata de trivializarlo demasiado. Esto tiene raíces en la historia. En los primeros días de la mecánica cuántica, se invocaba a las partículas y, en particular, a los fotones para explicar las "características de las partículas de la luz". En la visión moderna de la teoría cuántica de campos, esta imagen se reemplaza por lo que DanielSank describe en la pregunta vinculada.

Como tal, un fotón es complicado . No es a priori un paquete de ondas o una pequeña partícula puntual. La teoría del campo unifica ambas imágenes. Los campos de ondas de fotones reales son superposiciones de estas excitaciones fundamentales y pueden mostrar tanto el comportamiento del campo como el de las partículas. La respuesta a la siguiente pregunta del OP...

¿Cómo podemos entender la producción de pares si no entendemos qué es el fotón? ¿O el electrón? ¿O el campo electromagnético? ¿O todo lo demás?

... yace allí. Si quieres saber qué sucede con los objetos físicos reales, te estás alejando de los fotones. Los estados de un solo fotón en la naturaleza son raros, si no inexistentes.


Entonces, ¿qué es un fotón fundamentalmente ?

Citando de la pregunta:

[...] "el fotón es una excitación del campo de fotones". Eso no me dice nada.

Dice mucho, el formalismo matemático es muy claro y muchas personas han tratado de explicarlo en las respuestas aquí y en otros lugares.

[...] porque da la impresión de que los fotones siempre surgen y vuelan de un lado a otro ejerciendo fuerza. Este concepto también está en el artículo de Wikipedia sobre fotones. no es verdad Como dijo anna, las partículas virtuales solo existen en las matemáticas del modelo. Entonces, ¿quién puede decirme qué es un fotón real? [...]

El problema aquí es realmente la relación entre el formalismo matemático y la "realidad". Un fotón "real" no es una cosa, el fotón es una construcción matemática (que se describió anteriormente) y lo usamos (con éxito) para describir los resultados experimentales.


(1) cortesía de DanielSank.

Cuando Max Planck estaba trabajando para comprender el problema de la radiación del cuerpo negro, solo tuvo éxito cuando supuso que la energía electromagnética solo podía emitirse en forma cuantificada. En otras palabras, asumió que había una unidad mínima de luz que podía emitirse. Al asumir esto, por supuesto encontró mi = h v dónde h es la constante de Planck.

En 1905, Einstein tomó esto en serio y asumió que la luz existe como estas unidades fundamentales (fotones) con su energía dada por h v dónde v es la frecuencia de la radiación. Este fotón explicó muchos resultados experimentales y dio a la luz su dualidad onda-partícula.

Muchas cosas tienen un "trozo" mínimo: la longitud de Planck, por debajo de la cual la distancia pierde sentido, el quark, el gluón y otras unidades fundamentales de la materia, el tiempo de Planck (se supone que es la medida mínima del tiempo, es el tiempo necesario para que la luz para viajar en el vacío la distancia de una longitud de Planck).

Entonces, ¿qué es un fotón?

Es la "unidad" mínima de luz, la pieza fundamental. Diría que el átomo de luz, pero eso no transmite la imagen correcta. (¿El "quark" de la luz?)

También es importante recordar que no podemos "ver" un fotón, al igual que (bueno, incluso más) que no podemos "ver" un quark. Lo que sabemos sobre ellos es a partir de experimentos y cálculos, y como tal, no hay realmente una imagen física de ellos, como ocurre con la mayor parte de la mecánica cuántica.

Un fotón es solo una unidad de excitación de un modo electromagnético.
Podemos estar más cerca de ver un fotón de lo que piensas: nature.com/news/…
La mayoría de los físicos, incluidos los teóricos de cuerdas, desaprueban rotundamente que la longitud no tenga sentido por debajo de la longitud de Planck.
¿Cómo quiere decir que un fotón sea una unidad 'mínima' cuando puede tener cualquier energía en un rango continuo de energías?
@ user50229, un fotón es la unidad de energía más pequeña posible en una longitud de onda determinada.
@heather Si fija la longitud de onda, solo es posible una 'unidad' de energía, no hay un rango de energías para un fotón en una longitud de onda fija. Así que todavía no tiene sentido decir que el fotón es la unidad de energía más pequeña posible en una determinada longitud de onda. Es la única unidad.
@heather Creo que podría querer decir que un fotón transporta la menor cantidad de energía posible en una longitud de onda dada, ya que una mayor cantidad de energía requeriría más de una unidad (más fotones).
@ user50229, me disculpo si no fui claro, quise decir lo que dijiste (que un fotón transporta la menor cantidad de energía posible en una longitud de onda determinada).
Longitud de onda es solo un término conveniente. No hay ola. La distancia que recorre un solo fotón durante una oscilación se denomina longitud de onda. Un fotón con una longitud de onda de 500 nm en realidad tiene una frecuencia de más de 600 000 000 000 000 oscilaciones por segundo. A medida que avanza a la velocidad de la luz, una de esas 600 billones de oscilaciones se completa en 500 nm.
@heather: ¿Estaría de acuerdo en que solo podemos hablar de esta unidad de energía en el contexto de las interacciones, es decir, cuando el fotón se absorbe o se emite? Mientras la luz se propaga no sabemos si hay fotones.

Con cualquier concepto en física existe una dicotomía entre modelo y sistema físico. En la práctica nos olvidamos de esta dicotomía y actuamos como si modelo y sistema físico fueran lo mismo. Muchas respuestas a la pregunta "¿qué es un fotón?" reflejará esta identificación de modelo y sistema físico, es decir, un fotón es una partícula puntual ideal; un fotón es un cuanto de campo; un fotón esuna línea en un diagrama de Feynamn, etc. Estas definiciones de un fotón están profundamente arraigadas en los modelos. Nuestra predilección por identificar el modelo y el sistema físico se basa en la falsa suposición de que la intuición y las imágenes que desarrollamos para comprender el modelo se aplican igualmente al sistema físico. Nos convencemos de que la pequeña mota blanca que zumba a través del espacio tridimensional a la velocidad de la luz en nuestra cabeza es un fotón, cuando en realidad son imágenes asociadas con un modelo.

A la luz de esto, hay dos formas esenciales de responder a la pregunta "¿qué es un fotón?" La primera forma es referirse a un modelo y decir "el fotón es el concepto X en el modelo Y". Muchos usuarios han tomado esta ruta. La segunda forma es referirse a un experimento y decir "el fotón es lo que es responsable de este valor de datos". Tiendo a preferir esta ruta cuando respondo a la pregunta "¿qué es un ___?" porque evita la suposición de que el modelo y el sistema físico son idénticos. Aplicado al fotón diría "un fotón es un paquete de radiación electromagnética que satisface mi = h v y es el paquete más pequeño de radiación electromagnética".

Si no está satisfecho con ambos tipos de definición, no tiene suerte. Nuestros modelos siempre permanecerán en nuestras cabezas y el mundo físico que describen con tanta precisión siempre permanecerá fuera de nuestro alcance.

Este es un buen punto que probablemente será útil para cualquiera que esté desconcertado por esta variedad de descripciones.

Mi entrada:

Para un campo electromagnético libre o de interacción débil, que tiene radiación en alguna región a una frecuencia y energía definidas, hay una cantidad mínima de energía distinta de cero que se puede agregar o quitar del campo. Esa cantidad es un "fotón".

Ahora la letra pequeña:

  • Por supuesto, muchas otras respuestas aquí son más precisas, y creo que algunas también son perspicaces, pero tomé la pregunta como "explícalo como si fuera un niño, pero con la verdad", y traté de acercarme lo más posible. posible a esto.

  • Como otros han señalado, el fotón no siempre se usa de manera consistente, pero para prácticamente todos los usos que se me ocurren, la afirmación anterior es cierta (si cree que tiene un contraejemplo, indíquemelo). Digo "virtualmente" porque la única excepción que se me ocurre es el llamado "fotón virtual". Sin embargo, creo que esta terminología es ampliamente utilizada por los no expertos de todos modos y debe evitarse, o al menos debe discutirse por separado.

  • El acoplamiento "fuerte" versus "débil" tiene una definición estándar precisa entre los físicos, pero en realidad la transición entre fotones libres más excitaciones de materia a excitaciones fuertemente acopladas como polaritones ocurre sin problemas, y en ningún momento hay un cambio cualitativo brusco.

  • Experimentalmente, el requisito de "una frecuencia definida" a menudo se relajará ligeramente a "una frecuencia bien definida", porque cualquier fuente real de luz siempre tiene una amplitud espectral finita. Esta es una de las cuestiones que a veces provoca una ligera diferencia entre las nociones experimentalistas y teóricas de "fotón".

  • Esta definición, expresada en términos de la energía de una parte particular de un campo, es difícil al principio cuadrar con la imagen de "partícula similar a una bola de billar" que podría tener de los fotones como objetos discretos que vuelan y rebotan en las cosas. . Esto se debe simplemente a que esa imagen es, en muchos casos, gravemente defectuosa. En algunas situaciones muy específicas (tal vez la dispersión de Compton), es posible que pueda salirse con la suya. Sin embargo, a menudo es tan engañoso que probablemente sería mejor descartarlo por completo hasta que comprenda las condiciones bajo las cuales es válido aproximadamente, lo cual es un tema sutil que merece una discusión completamente separada. La mayoría de las veces, los fotones no son en absoluto como pequeñas "bolas de luz de billar".

Estoy totalmente de acuerdo con la respuesta de flippiefanus . En primer lugar, un fotón es un concepto útil introducido para describir fenómenos para los que no tenemos un enfoque intuitivo y, aparte de eso, no sabemos realmente qué es un fotón. Aunque es cierto, esto no es particularmente satisfactorio. Lo que quiero agregar a su respuesta es por qué se introdujo el concepto de fotón.

Durante mucho tiempo se discutió si la luz es una partícula o una onda. Newton apoyó y desarrolló gran parte de la "teoría corpuscular de la luz" . Su argumento más fuerte fue que la luz viaja en línea recta, mientras que las ondas tienden a dispersarse espacialmente. Huygens, por otro lado, argumentó que la luz es una onda. La teoría ondulatoria de la luz podría explicar fenómenos como la difracción, que la teoría corpuscular no pudo explicar. Cuando Young realizó su famoso experimento de la doble rendija , que mostró patrones de interferencia como los que se conocen de las ondas de sonido o de las ondas de agua, la cuestión pareció resolverse de una vez por todas.

[Una nota al margen interesante: las ondas necesitan un medio para propagarse, pero la luz también se propaga a través del vacío. Esto condujo a la postulación del llamado éter , un medio que se suponía que impregnaba todo el espacio. Las propiedades de este éter, sin embargo, eran contradictorias y no se encontró evidencia experimental de ello. Esto jugó un papel en el desarrollo de la teoría de la relatividad, pero esa es otra historia]

Luego, alrededor de 1900, Max Planck pudo describir correctamente el espectro de un cuerpo negro con lo que al principio pensó que era solo un truco matemático: para sus cálculos asumió que la energía se irradia en pequeñas porciones, en lugar de continuamente, como usted supondría si la luz fuera una onda. El espectro del cuerpo negro era una de las preguntas sin resolver más importantes en ese momento y su explicación fue un gran avance científico. En consecuencia, su método recibió mucha atención.

Poco después, Einstein utilizó el método de Planck para explicar otro problema no resuelto de la física: el efecto fotoeléctrico . Nuevamente, este fenómeno podría describirse si la luz se imagina como pequeños paquetes de energía. Pero a diferencia de Planck, Einstein consideró estos paquetes de energía como una realidad física, a los que luego se denominó fotones .

Este neologismo estaba a todas luces justificado, porque para entonces ya estaba claro que los fotones tenían que ser algo más que pequeñas bolas de billar como imaginaba Newton. A veces exhibe propiedades ondulatorias que no pueden explicarse con partículas clásicas, a veces exhibe propiedades similares a partículas que no pueden explicarse con ondas clásicas.

Esto es lo que sabemos:

  • La luz se emite en un número discreto de paquetes. Esto significa que hay entidades contables de luz (a las que hoy llamamos fotones). Esta afirmación es probablemente la más fundamental de la idea del fotón. También sería mi "respuesta de una oración" si alguien me preguntara qué es un fotón.

  • Llevan propiedades físicas como energía y cantidad de movimiento y pueden transferirlas entre objetos físicos (p. ej., cuando la radiación térmica se absorbe y calienta un cuerpo).

  • Dado que los fotones tienen un momento, también deben tener masa .

  • Más tarde también se demostró que los fotones tienen otras propiedades como el espín .

  • Se propagan en línea recta sin necesidad de un medio (se demostró que el éter que mencioné antes no existe).

Todas estas propiedades se asocian comúnmente con las partículas. Como mínimo, muestran que un fotón es algo (en el sentido de que una colección de propiedades físicas normalmente califica como una cosa ). Pero un fotón también tiene propiedades diferentes a las de los objetos clásicos:

  • Cuando los fotones se propagan muestran difracción , refracción e interferencia .

  • La energía y el momento de un fotón corresponden a la longitud de onda y la frecuencia de la luz, que gobiernan el comportamiento de interferencia y difracción.

La conclusión es que un fotón no es ni una onda ni una partícula, sino un objeto cuántico que tiene propiedades tanto de onda como de partícula. En general, se puede decir que las propiedades similares a las partículas son dominantes cuando observas

  • pequeñas cantidades de fotones

  • sus interacciones con la materia (como el proceso de producción de pares que mencionaste)

  • altas energías

mientras que las propiedades ondulatorias son dominantes cuando miras

  • gran cantidad de fotones

  • su propagación en el espacio

  • bajas energías

Creo que esto es lo más lejos que se puede llegar con las analogías clásicas. Un fotón es lo que nos dicen sus propiedades y su comportamiento, y todo lo demás es solo una analogía incompleta. Personalmente, me gusta imaginar los fotones (al igual que con cualquier visualización, esto no es correcto, pero funciona bien en muchas situaciones y ayuda a controlarlo) como partículas pequeñas, duras y discretas que se mueven en el espacio como lo harían las ondas.

No estoy de acuerdo con que "la luz se emita en un número discreto de paquetes". Según tengo entendido, es la medida la que 'colapsa' la función de onda del fotón para que los fotones aparezcan como clics discretos en el contador de fotones, etc. En otras palabras, durante la propagación cuántica de un estado de salida de un átomo, muchos estados de fotones adquieren amplitud. . Estos, sin embargo, se enredan en los resultados del dispositivo de medición de tal manera que la discreción aparece debido a la decoherencia. Yo diría: "la luz se mide en paquetes discretos".
"Dado que los fotones tienen un impulso, también deben tener masa". Disparates. Una comprensión relativista del impulso es bastante esencial para una entidad intrínsecamente relativista, ¿no?
La energía de la luz no es un valor cuantificado. La energía de la luz de una frecuencia específica dada es un valor cuantificado, basado en la fórmula w=hv. la energía total de la luz de frecuencia=v debe ser N veces de hv. Pero esto es evidente y parece no tener mucho sentido. Tomamos cualquier cantidad de cosas como inicial, el valor total debe ser N veces esa unidad.
@dmckee Masa relativista.

La radiación se emite cuando un electrón se desacelera y se absorbe cuando se acelera según la conocida fórmula de Larmor. Esta radiación es un campo electromagnético continuo. Dentro del átomo, los electrones cambian de órbita y se aceleran rápidamente en el proceso, lo que da como resultado la emisión y absorción de radiación tan rápido que aparece como líneas de espectros. Pero aparte de las líneas espectrales, los átomos y las moléculas emiten a muchas otras frecuencias debido a los diversos movimientos de oscilación en ellos y la correspondiente aceleración/desaceleración que lo acompaña. Estas frecuencias de fondo serían naturalmente de menor frecuencia que la de los espectros de línea para el mismo sistema. La radiación de Cherenkov es quizás la más cercana a un espectro continuo.

Por eso la radiación de toda la materia se compone de líneas espectrales nítidas sobre un fondo de radiación continua. El fotón es la unidad de energía de radiación intercambiada entre electrones enlazados (no libres). Es como la moneda del dinero... y como la moneda, los fotones no son todos de la misma denominación/energía. La fórmula E=nhf da n como el número de fotones intercambiados de frecuencia f que dan como resultado la energía E. Pero como f es variable y ni siquiera discreta, E no es discreta, pero el fotón sí lo es.

El fotón también se describe como un paquete de energía. Esto es correcto, pero solo significa la energía mínima (n=1) de un cierto color que se puede intercambiar en una determinada interacción entre átomos. Normalmente la energía de las ondas es directamente proporcional al cuadrado de su amplitud y nada que ver con la frecuencia. Para conciliar esto con la definición del fotón, muy necesaria para la interacción nuclear, se introduce el número de fotones n para compensar la fórmula original E=hf. Tenga en cuenta que, si bien un fotón es un paquete de energía, la cantidad de energía en el paquete puede variar. Un solo fotón azul tiene mucha más energía que un fotón rojo, por ejemplo. Los rayos gamma tienen el mayor contenido de energía debido a su mayor frecuencia.

Si lee un libro sobre fotónica, encontrará que la palabra fotón aparece en casi todas las líneas. Esto muestra cuán importante es el concepto de fotón, a pesar de su definición y uso inusuales y un poco confusos.

Según tengo entendido estamos de acuerdo:

Usamos la palabra fotón cuando de alguna manera observamos que un objeto material cambia el contenido de energía, el impulso y el giro y no hay otro objeto material en las proximidades directas para interactuar.

Sabemos que hay fuerza magnética y fuerza eléctrica y que estas fuerzas se pueden unificar a la fuerza electromagnética y el desarrollo espacial y temporal de esas fuerzas permite ondas electromagnéticas muy parecidas a otras ondas que se pueden observar directamente como ondas de agua.

Del agua y del aire sabemos que pueden existir ciertas excitaciones como ondas planas o circulares. Las formas de onda especiales son vórtices o solitones más generales.

Sabemos que los solitones pueden existir como excitaciones cuantificadas como en los superfluidos. Entonces, existe una cierta probabilidad de que tales solitones puedan ser soluciones de las Ecuaciones de Maxwell. Eso permitiría imaginar lo que realmente "es" un Fotón.

Mientras escribía, tuve la idea de buscar en Google "fotón solitón". An encontré este documento abierto como punto de partida para mí: Photons as solitons y allí: acceso directo

PD: no rebaje esta respuesta, ya que es un trabajo en progreso y reaccionaré ante cualquier comentario para ser más claro. Y no puedo comentar a otros responder en este momento. Gracias

Esta respuesta no tiene en cuenta la gravedad… Por ejemplo. Deja que el cuerpo tenga una cavidad. En esta cavidad el cuerpo irradia fotones y absorbe fotones. Hay un intervalo de tiempo entre la emisión y la absorción. La masa del cuerpo consiste en una masa del material del cuerpo y la energía de los fotones en esta cavidad para un observador externo. ¿Existen fotones en el vacío? ¿Son sus parejas una fuente de gravedad? ¿Los fotones influyen en el espacio-tiempo?
@Imyaf: la pregunta es bastante simple y las respuestas dadas son muy complejas. Veo las cosas materiales como condensados ​​de energía localizados. Como sabemos que la materia puede desintegrarse en reacciones nucleares emitiendo fotones, los fotones atrapados deberían contribuir a la masa de la trampa. Pero esto solo se puede medir para fotones de alta energía (gamma).

Un amigo me preguntó esto en la universidad, y esto es más o menos lo que le dije.

L os experimentadores estaban descifrando el comportamiento de los fenómenos eléctricos y magnéticos a fines del siglo XVIII y principios del XIX, y para mediados del siglo XIX todo se estaba juntando. James Clerk Maxwell dio los "toques finales" a las ecuaciones que describían los fenómenos eléctricos y magnéticos (clásicos).

Una de esas ecuaciones (ley de Faraday) describe cómo un campo magnético cambiante puede inducir una corriente eléctrica , mientras que otra ecuación (ley de circuito de Ampère) describe cómo una corriente eléctrica puede inducir un campo magnético .

Así que piense en los electrones, tienen carga eléctrica, si "sacudimos" uno correctamente , podemos crear un campo eléctrico cambiante, que induce un campo magnético cambiante, que induce un campo eléctrico cambiante, y así sucesivamente... Estas pequeñas ondas de los campos eléctrico y magnético, induciéndose uno al otro, eso es lo que son los fotones .

En algún momento leí un relato fascinante de Maxwell, sobre cómo una vez que había resuelto las ecuaciones para describir el electromagnetismo, observó que podían usarse para derivar una ecuación de onda. Las ecuaciones de onda tienen una constante que describe la velocidad de propagación de las ondas y su ecuación de onda derivada tenía 1 m ϵ para esa constante (con m siendo la permeabilidad y ϵ la permitividad).

Si no recuerdo mal, estos valores podrían medirse mediante experimentos con electricidad. Algo así como enviar una cantidad conocida de corriente a través de dos cables paralelos, generarán una fuerza magnética que separará los cables (si la corriente está en la misma dirección, y los juntará si está en direcciones opuestas, ¿creo?). Medir la fuerza resultante puede decirle el valor (supongo que solo uno de ellos).

Así que estos valores se habían medido, y conectándolos a Maxwell se obtuvo algo cercano a la velocidad de la luz, que los experimentadores habían estado midiendo con mayor precisión en ese momento (especialmente en 1849 y 1862). Y esta fue la primera vez que alguien (Maxwell) pudo darse cuenta de que la luz era una especie de fenómeno electromagnético. [Mirando hacia arriba veo que en realidad , Wilhelm Eduard Weber y Rudolf Kohlrausch en 1855 habían notado las unidades de m y ϵ podían producir una velocidad, y las midieron experimentalmente y llegaron a un número muy cercano a la velocidad de la luz, pero no dieron ese salto final de lógica, que Maxwell hizo en 1861.] (Del artículo de wikipedia History of Maxwell's ecuaciones )

No soy un experto, pero mi impresión es que Maxwell también notó que sus ecuaciones parecían incompletas, porque sugerían que la velocidad de la luz permanecía constante independientemente de la velocidad del observador o del emisor. Es común que la gente piense que el trabajo de Einstein sobre la relatividad especial estaba resolviendo el famoso resultado nulo de los experimentos con interferómetro de Michelson y Morley en busca de un éter luminífero, pero Einstein en realidad estaba abordando esta invariancia indicada por las ecuaciones de Maxwell, si entiendo bien.

(Tenga en cuenta que ha pasado mucho tiempo desde que conté la mayor parte de esto, y solo tengo una licenciatura en matemáticas y física, y no he usado nada de este conocimiento en mucho tiempo, por lo que es posible que yo' Me equivoco en algunos detalles, pero creo que la esencia general está bastante cerca).

Antes incluso de intentar preguntar qué es un fotón "exactamente", tenemos que preguntarnos: ¿existen los fotones?

Puedes recorrer un largo camino creyendo que no lo hacen. Los átomos, las moléculas y los cristales tienen estados discretos que determinan la naturaleza cuántica de la materia, por lo que emiten y absorben cuantos de energía mientras que la entidad en sí misma puede ser continua, al igual que el vino es una entidad continua que solo se cuantifica en 70 cl debido a las botellas que contiene. se vende en. La mecánica cuántica utiliza el campo EM clásico. Las líneas onduladas en los diagramas de Feynman, a menudo vagamente llamadas fotones, son solo una notación gráfica para los términos en una expansión de perturbación.

Sin embargo, sigue existiendo un problema: ¿cómo es que los fotones se absorben en reacciones muy locales? ¿Cómo puede ser absorbida por un solo átomo una onda electromagnética clásica extendida? Para mí, una interpretación sensata de estos fenómenos es que el campo EM describe la probabilidad de que se produzca una absorción/emisión.

Por esta razón, ahora estoy convencido de que existen fotones discretos y que la ecuación de onda subyacente a las ecuaciones de Maxwell es una ecuación de onda cuántica relativista que describe partículas cuánticas sin masa, al igual que las ecuaciones de Schrödinger, Dirac y Klein-Gordon describen partículas cuánticas masivas. La ecuación de onda electromagnética en mi interpretación es una ecuación de Klein-Gordon sin masa que describe las partículas cuánticas conocidas como fotones.

Esto no responde a la pregunta de qué es exactamente un fotón. Propone una respuesta a la pregunta anterior si existen fotones.

Esta es la gran pregunta "zen" de la física durante siglos, gracias por preguntarla. Otras respuestas son buenas/aceptables, esta (riesgosa para la reputación, pero sincera/detallada) adopta, de alguna manera, un ángulo/enfoque radicalmente diferente. Otras respuestas miran hacia el pasado, esta intentará hacer lo casi imposible de anticipar el futuro de una manera visionaria pero = --- todavía científicamente fundamentada. En otras palabras, las partes cuestionables pueden considerarse hipotéticas, es decir, hipótesis bajo consideración, pero todas cuidadosamente respaldadas por hallazgos de investigación actuales/sólidos (en algunos casos, muy recientes).

La historia del fotón tiene un dramático "aspecto de ciegos y elefantes" que atraviesa muchos siglos.[1] La naturaleza de onda versus partícula de la luz incluso se debatió en la época de Newton en el siglo XVII, ahora cerca de hace ~ 4 siglos y las unidades de luz o partículas se denominaron "corpúsculos" en contraste con la teoría de onda de Huygens. [2] La teoría de Newton dominó durante algo así como un siglo sobre el último "en parte debido al gran prestigio de Newton", a pesar de que Huygens se formuló casi / inicialmente al mismo tiempo. Esto muestra un ejemplo del efecto "inverso" de la reputación humana en el pensamiento científico de la época.

Los experimentos recientes de Lacour-Ott son un gran avance y muestran "un modelo de medición cuántica basado en detectores y determinista localmente".[3][4] Este es un hallazgo sorprendente que aún no ha sido ampliamente considerado. Demuestra que un formalismo mecánico cuántico completo puede surgir en el análisis de meros sistemas clásicos. Entonces, esto pone en serio cuestionamiento las afirmaciones de casi un siglo de que la mecánica cuántica es intrínsecamente diferente de la mecánica clásica, ahora vista no solo como un sistema de creencias, sino como un dogma virtual del campo. Hay muchos otros desarrollos recientes que ponen grietas/abolladuras en esta armadura larga y parecen forzar una reevaluación/reconsideración[7] (pero este seguramente será un proceso largo y apenas comienza).

Las nuevas teorías se están comparando con la mecánica de Bohm, pero tienen aspectos claramente diferentes y nuevos, y no deben descartarse instintivamente como refutadas. Una de las encuestas más completas hasta ahora es la de Bush.[5] ¡Está recientemente respaldado por experimentos! [6]

Entonces, ¿cómo es esto posible conceptualmente/teóricamente? Una novedad sorprendente es que la ley probabilística de Borns en la mecánica cuántica puede surgir en los sistemas clásicos. Véase, por ejemplo, Qiaochu Yuan, "Probabilidad no conmutativa finita, la regla de Born y el colapso de la función de onda" [8] y otro análisis mucho más detallado de los detectores proviene de Khrennikov y "PCST", "Prequantum Classical Statistical Field Theory", lo que también se conoce como teorías más o menos semiclásicas.[9][10]

[9] Habla sobre un detector que descarta energía donde la energía entrante no coincide con el umbral de energía del detector (p9) y el detector "come una porción de energía" (p10). Llamemos a esto un detector disipativo. otro concepto similar en la medición es el tiempo muerto del detector[10p5] donde "el detector no puede interactuar con un pulso entrante".

Parece que estos conceptos son similares a un estudio muy sofisticado/comprehensivo del teorema de Bell donde el sistema de señalización puede tener los llamados eventos de "aborto" y que encuentra que las versiones más estrictas de las desigualdades de Bell no son violadas por los experimentos actuales.[11][13 ]

Estos son similares a la "laguna de muestreo"[12] que no es necesariamente lo mismo que las llamadas lagunas de eficiencia, porque la primera aún puede persistir incluso cuando la eficiencia del detector se mide al 100%.

Exploremos el concepto de un detector disipativo con más cuidado y cómo se vería teóricamente. Considere el siguiente esquema. Un frente de onda único esférico viaja a través del espacio. Ahora imagínelo pasando por el detector. el detector puede estar en la región de tiempo muerto y no detectará el frente de onda. O puede que lo detecte. Esta es la naturaleza probabilística de la luz. Parece que posiblemente el tiempo muerto no se puede reducir a cero como una ley física relacionada/similar al principio de incertidumbre de Heisenberg.

Otra forma de decir todo esto es que los detectores perfectos no existen. Los únicos detectores que tenemos están hechos de átomos, también conocidos como partículas. El misterio del fotón finalmente se desentraña. Un fotón es una interacción (probabilidad) entre un frente de onda y un dispositivo de medición, es decir, un átomo u otra partícula. La interacción sólo puede ser referenciada a posteriori y no a priori . En otras palabras, incluso un detector hecho de un solo átomo tendrá esta propiedad de tiempo muerto y disipación de energía. Así que ahí también tenemos alguna interpretación para las llamadas partículas virtuales .

Otros pueden cuestionar "un frente de onda único esférico" que viaja por el espacio. Exactamente esa imagen ahora está respaldada por un nuevo modelo de espacio-tiempo descrito exhaustivamente por Tenev/Horstemeyer, el "tejido del espacio-tiempo".[14] no parecen considerar mucho el tensor de estrés EM, pero una generalización obvia de su trabajo es que las ondas EM son ondas s en el tejido del espacio-tiempo.

Un experimento bastante sencillo que demuestra estas ideas es el efecto HBT. Imagine una línea de detectores a la misma distancia exacta de una fuente de "fotón único" como una forma sencilla de aumentar la sensibilidad de detección del frente de onda. La idea de una fuente de "fotón único" puede visualizarse mejor como una fuente de "frente de onda único". A medida que el frente de onda pasa a través de los detectores, cada detector puede hacer clic o no. Si hubo algún clic, hubo un frente de onda. Si ninguno hace clic, es posible que haya pasado el frente de onda, pero es posible que todos hayan estado en su período de tiempo muerto "sin respuesta". La matriz combinada general detectará el frente de onda con mayor precisión.

Este efecto ya se observa pero no se interpreta bajo este punto de vista. Se llama (anti) agrupamiento de fotones en la literatura. Muchos otros efectos se malinterpretan actualmente bajo la niebla/neblina de nuestra teoría actualmente nublada. Tomará mucho tiempo volver a trabajarlo todo. Pero tales reelaboraciones no son desconocidas en la historia de la ciencia, aunque tienden a tratarse de eventos que ocurren una vez cada siglo y literalmente conducen a/requieren, por ejemplo, que se reescriban los libros de texto (¡pero no todos a la vez!).[17] No se pueden cronometrar exactamente (análogamente a los terremotos ) e incluso son difíciles de reconocer en el medio, pero algunos signos (referencias recopiladas, por ejemplo, también [18], muchos otros no citados debido a limitaciones de espacio/formato, cita siguiente, etc.) están presentes actualmente y parece que estamos atrasados ​​para uno.

“Desearía que las personas que estaban desarrollando la mecánica cuántica a principios del siglo pasado tuvieran acceso a estos experimentos”, dijo Milewski, “porque entonces toda la historia de la mecánica cuántica podría ser diferente”.[7]

[1] ciegos y elefantes / wikipedia

[2] Teoría corpuscular de la luz / wikipedia

[3] Un modelo de medición cuántica basado en detectores y determinista localmente / La Cour

[4] Computadora cuántica emulada por un sistema clásico / physorg

[5] Hidrodinámica de onda piloto / Bush

[6] nuevo soporte para vista cuántica alternativa / Wolchover

[7] ¿Hemos estado interpretando mal la mecánica cuántica todo este tiempo? / Wolchover

[8] Probabilidad no conmutativa finita, la regla de Born y el colapso de la función de onda Qiaochu Yuan

[9] Regla de Born a partir de mediciones de señales clásicas mediante detectores de umbral debidamente calibrados / Khrennikov

[10] Teoría de campos estadísticos clásicos precuánticos: simulación de probabilidades de detección de fotones con la ayuda del movimiento browniano clásico / Khrennikov

[11] Desigualdades robustas de Bell desde la complejidad de la comunicación / La Plante

[12] Lagunas en los experimentos de prueba de Bell / wikipedia

[13] Violación de la desigualdad de Bell sin lagunas utilizando espines de electrones separados por 1,3 kilómetros / Hensen et al

[14] La mecánica del espacio-tiempo: una perspectiva de mecánica sólida sobre la teoría de la relatividad general / Tenev, Horstemeyer

[15] Tensor de energía de estrés EM / wikipedia

[16] efecto HBT / wikipedia

[17] Cambio de paradigma / wikipedia

[18] EmQM13: Conferencia / Actas de Mecánica Cuántica Emergente 2013

La forma más fácil de entender un fotón es que es una partícula que obedece a la mecánica cuántica más que a la mecánica clásica. En mecánica cuántica, las partículas no tienen una posición, pero podemos calcular la probabilidad de dónde se encontrará la partícula (o, en el caso de un fotón, dónde será aniquilada). Como dijo Dirac

“En el caso general, no podemos hablar de que un observable tenga un valor para un estado particular, pero podemos… hablar de la probabilidad de que tenga un valor específico para el estado, es decir, la probabilidad de que se obtenga este valor específico cuando se hace una medida de lo observable”.

El cálculo de las probabilidades cuánticas es necesariamente diferente al de las probabilidades clásicas, porque la mecánica cuántica describe procesos realmente indeterminados, mientras que en la teoría estándar de la probabilidad los resultados están determinados por incógnitas. Por razones profundas y sutiles en los fundamentos matemáticos de la mecánica cuántica (que generalmente no se tratan en los libros de texto estándar), la interpretación de la probabilidad requiere que el cálculo siga las leyes de la mecánica ondulatoria, creando la ilusión de que las partículas tienen propiedades ondulatorias.

He mostrado el argumento matemático en The Mathematics of Gravity and Quanta

Citas pertinentes de Einstein :

  • Todos los cincuenta años de cavilaciones conscientes no me han acercado más a responder la pregunta: “ ¿Qué son los cuantos de luz? Por supuesto, hoy en día todo bribón cree saber la respuesta, pero se engaña a sí mismo.

  • Por el resto de mi vida reflexionaré sobre lo que es la luz .

¿Hay algo más en el fotón que los cambios en las propiedades de su emisor y absorbente solos? (Respuesta reexpresada 6-11-16)

Después de un poco más de respuestas negativas que positivas, y también de una corrección útil, en general es hora de dejar de cavarme en un agujero.

No obstante, revisando la literatura reciente, quisiera terminar llamando la atención de los críticos sobre un artículo reciente en Quantum Stud.: Math. Fundar. (2016) 3: 147–160 (copia adjunta Rashkovskiy (2016) ) que también argumenta a favor de la inexistencia del fotón, pero a través del enfoque semiclásico. Aunque aborda el asunto con mucha mayor profundidad y detalle de lo que yo podría lograr, creo que es consistente con la opinión de que, en última instancia, las características del fotón podrían buscarse mejor en las propiedades del emisor y el absorbente, en lugar de atribuirse a un fotón ficticio… ¿o tiene alguna diferencia si lo consideramos o no como 'real'?

Como los comentarios indicaron que parte de mi argumento es defectuoso, se han eliminado las referencias a "puntos coincidentes de espacio-tiempo" y "interacción de contacto" en el espacio de Minkowski.

Sin embargo, las referencias en la contribución anterior de Arthur Neumaier incluyen casos de físicos eminentes que argumentan la inexistencia de fotones:

  • PCW Davies: "En su provocativamente titulado artículo 'Las partículas no existen', Paul Davies avanza varias dificultades profundas para cualquier concepción de partícula convencional del fotón"

  • Se dijo que el ganador del premio Nobel Willis Lamb "sostenía que los fotones no existen".

Ofrezco un argumento elemental , basándome en las conferencias de Feynman (Vol 1, final de la sección 17-2 en la página 17-4),

"En nuestro diagrama de espacio-tiempo, por lo tanto, tendríamos una representación similar a esta: a 45° hay dos líneas (en realidad, en cuatro dimensiones serán "conos", llamados conos de luz) y los puntos en estas líneas son todo a intervalo cero desde el origen.Donde la luz va desde un punto dado siempre está separado de él por un intervalo cero, como vemos en la ecuación (17.5).Por cierto, acabamos de probar que si la luz viaja con velocidad c en un en otro sistema, viaja con velocidad c en otro, pues si el intervalo es el mismo en ambos sistemas, es decir, cero en uno y cero en el otro, entonces afirmar que la velocidad de propagación de la luz es invariable es lo mismo que decir que la el intervalo es cero".

en el que afirma que los puntos en el cono de luz están a una distancia cero del origen y entre sí. Por supuesto, él quiere decir cero 4 distancias en el espacio-tiempo, como resultado de la métrica de Minkowski que requiere una resta entre el intervalo de 3 espacios yc veces el intervalo de tiempo.

Entonces, se puede argumentar que los fotones no tienen una existencia independiente en el espacio-tiempo: "aparecen" solo como una pérdida, por emisión de una fuente, y una ganancia por absorción [se elimina el resto de la oración original].

Desde este punto de vista, los fotones son, en cierto sentido (como otros han mencionado en respuestas más sofisticadas) simplemente ficciones convenientes de las matemáticas que explican lo que sucede en la transferencia de energía observada entre el emisor y el absorbente, a través de un intervalo de tiempo y espacio tridimensional que hacemos un puente usando las matemáticas que describen los movimientos de onda. Esto sugiere que las características del fotón (por ejemplo, el espín) deben buscarse en las propiedades del emisor y el absorbente , en lugar de atribuirse al fotón ficticio...

Respondiendo a los votantes negativos (NB, también ha habido un par de votantes positivos) ¿Puede indicar exactamente dónde cae la idea? Simplemente estoy desconcertado por lo que, en mi relativa ignorancia de químico, parece ser la clara implicación de SR de que el paso de lo que describimos como un fotón es solo una interacción del emisor y el absorbente que, según Feynman, están en directa "4 -contacto” – ningún intervalo finito de espacio-tiempo los separa. Mi pregunta es un intento de aclarar este aparente rompecabezas. Gracias
La distancia del espacio-tiempo cero no es lo mismo que los "puntos coincidentes", la "métrica" ​​de Minkowski de manera crucial no induce una métrica adecuada en el sentido matemático de que dos puntos con distancia cero son iguales, por lo que la parte sobre la "interacción de contacto" es simplemente no -sensato Hay infinidad de puntos distintos que están a un intervalo de espacio-tiempo cero entre sí, y esta no es una relación transitiva (considere la intersección de dos conos de luz de puntos espacialmente separados: la intersección está a una distancia cero de ambos, pero los dos puntos son no).
@igael Gracias por la edición - mejora la presentación
@ACuriousMind - Gracias. ¿Dónde puedo encontrar (o para qué sería una palabra clave) algo sobre la diferencia entre la "métrica" ​​de Minkowski y lo que usted llama una métrica adecuada? Recibí otra comunicación en la que me desengañaron de la noción de una interacción de contacto, por lo que agradezco la confirmación (suponiendo que no sea también "UM")
Lo que yo llamo una métrica "adecuada" sería una métrica en el estricto sentido matemático . Se ajusta exactamente a nuestra intuición de la distancia, en particular el hecho de que dos puntos con distancia métrica cero son el mismo punto, es decir, "en contacto". La noción SR/GR de "métrica" ​​es la de un tensor métrico , que da una métrica "adecuada" solo en el caso de Riemann, pero SR/GR es solo lorentziano/pseudo-riemanniano.
@ACuriousMind Entonces, la raíz de mi error está en confundir las propiedades de la métrica de Minkowski con las de una estrictamente matemática, al menos con respecto a una "interacción de contacto". Supongo que son los cambios de signo en la firma los que son el problema. Aun así, todavía necesito convencerme de que necesitamos el fotón como algo más que una ficción conveniente que une el emisor con el absorbente. Gracias otra véz.
@ user40110 Además de SR, ¿qué te hace creer que los fotones pueden no existir en absoluto? ¿La ecuación mi = h F o mi = 2.210 X 10 42 C F hacerte creer esto? ¿Qué parte de esta ecuación haría creer a alguien que existe una partícula?
@Inquisitive Responder a seguir cuando me hayan aconsejado cómo publicar un archivo (no tengo dónde publicarlo con un enlace), pero creo que es demasiado largo para publicarlo como un comentario abierto (~2500 palabras).
@ user40110 Esperaré ansiosamente tu publicación. O bien, podría truncarlo manteniendo los puntos principales.
@Inquisitivo Al no poder escribir una respuesta adecuada a su primera pregunta en el espacio permitido, la publiqué como PDF al comienzo del hilo para la pregunta correspondiente en ResearchGate en researchgate.net/post/… que comencé en para obtener más discusión, antes de que otros comenten aquí. El archivo se centra en su primera pregunta: "Aparte de SR, ¿qué le hace creer que los fotones pueden no existir en absoluto?" como una revisión de mi posición y argumentos anteriores. Mis respuestas a la segunda y la tercera pregunta son No y Probablemente_ninguna .
@ Inquisitivo Entonces, ¿pudiste ver la respuesta que dijiste que "esperarías ansiosamente"?
25-5-18 Agradecería tener la oportunidad de aprender del voto negativo de hoy...